E3C2 – Spécialité maths – Suites – 2020

Suites

E3C2 – 1ère

En France métropolitaine, 2018 a été l’année la plus chaude d’après les relevés météorologiques. La température moyenne y a été de $14$ °C; elle a dépassé de $1,4$ °C la normale de référence calculée sur la période 1981-2010. (Source : site Météo France)

  1. Pour modéliser la situation, on considère l’année 2018 comme l’année zéro et on suppose que cette hausse moyenne de 1,4°C par an se poursuit chaque année. Pour tout entier
    naturel $n$, on note alors $T_n$ la température moyenne annuelle en France pour l’année 2018$+n$.
    a. Quelle est la nature de la suite $\left(T_n\right)$ ainsi définie ? On donnera son premier terme et sa raison.
    $\quad$
    b. On considère qu’au-delà d’une température moyenne de $35$°C les corps ne se refroidissent pas et il devient insupportable pour les humains de continuer à habiter cette région que l’on qualifie alors d’inhabitable. Selon le modèle considéré, en quelle année la France deviendrait-elle inhabitable pour les humains ? Justifier.
    $\quad$
  2. À cause du réchauffement climatique, certaines régions risquent de connaître une baisse de $10 \%$ par an des précipitations moyennes annuelles mesurées en millimètres (mm).
    Dans une région du nord de la France, les précipitations moyennes annuelles étaient de $673$ mm en 2018. On considère l’année 2018 comme l’année zéro et on suppose que cette baisse de $10 \%$ par an se poursuit chaque année. Pour tout entier naturel $n$, on note $P_n$ les
    précipitations annuelles moyennes en mm dans cette région pour l’année 2018$+n$.
    a. Quelle est la nature de la suite $\left(P_n\right)$ ainsi définie ? On donnera son premier terme et sa raison.
    $\quad$
    b. Pour tout entier naturel $n$, exprimer $P_n$ en fonction de $n$.
    $\quad$
    c. On donne le programme Python suivant :
    $$\begin{array}{l}
    \text{def precipitations(J):}\\
    \hspace{0.5cm}\text{I=673}\\
    \hspace{0.5cm}\text{n=0}\\
    \hspace{0.5cm}\text{while I > J:}\\
    \hspace{1cm}\text{I = 0.9*I}\\
    \hspace{1cm}\text{n = n+1}\\
    \hspace{0.5cm}\text{return n+2018}\end{array}$$
    L’exécution de « $\text{precipitations(300)}$ » renvoie la valeur $\text{2026}$ . Que représente cette valeur pour le problème posé ?
    $\quad$

$\quad$

Correction Exercice

  1. a. Pour tout entier naturel $n$ on a donc $T_{n+1}=T_n+1,4$.
    La suite $\left(T_n\right)$ est donc arithmétique de raison $1,4$ et de premier terme $T_0=14$.
    $\quad$
    b. Pour tout entier naturel $n$ on a donc $T_n=14+1,4n$.
    On veut par résoudre l’inéquation :
    $\begin{align*} T_n>35 &\ssi 14+1,4n>35 \\
    &\ssi 1,4n>21 \\
    &\ssi n>15\end{align*}$
    Selon ce modèle, c’est à partir de l’année 2034 que la France deviendrait inhabitable pour les humains.
    $\quad$
  2. a. Pour tout entier naturel $n$ on a :
    $\begin{align*} P_{n+1}&=\left(1-\dfrac{10}{100}\right)P_n\\
    &=0,9P_n\end{align*}$
    La suite $\left(P_n\right)$ est donc géométrique de raison $0,9$ et de premier terme $P_0=673$.
    $\quad$
    b. Pour tout entier naturel $n$ on a donc $P_n=673\times 0,9^n$.
    $\quad$
    c. Selon ce modèle c’est à partir de l’année 2026 que les précipitations moyennes annuelles seront inférieures ou égales à $300$ mm.
    $\quad$

[collapse]

$\quad$

Les sujets proviennent de la banque nationale de sujets sous licence

 

E3C2-Spécialité maths – Suites – 2020

Suites

E3C2 – 1ère

Durant le mois de janvier 2020, une entreprise produit $2~500$ flacons de parfum ce qui correspond exactement au nombre de flacons commandés. Le propriétaire de l’entreprise décide d’augmenter chaque mois la production de $108$ flacons et il espère que le nombre de flacons commandés augmentera chaque mois de $3,8 \%$.
On considère la suite $\left(f_n\right)$ où pour tout entier naturel $n$, $f_n$ modélise le nombre de flacons produits lors du mois de rang $n$ après janvier 2020 ; ainsi $f_0$ est le nombre de flacons produits en janvier 2020, $f_1$ le nombre de flacons produits en février 2020, etc.
De la même manière, on considère la suite $\left(c_n\right)$ où pour tout entier naturel $n$, $c_n$ modélise le nombre potentiel de flacons commandés lors du mois de de rang $n$ après janvier 2020. On a donc $f_0=c_0=2~500$.

  1. Déterminer, en expliquant les calculs effectués, le nombre de flacons produits et le nombre potentiel de flacons commandés en février 2020.
    $\quad$
  2. Déterminer la nature des suites $\left(f_n\right)$ et $\left(c_n\right)$.
    $\quad$
  3. Exprimer, pour tout entier $n$, $f_n$ et $c_n$ en fonction de $n$.
    $\quad$
  4. On admet que, selon ce modèle, au bout d’un certain nombre de mois le nombre potentiel de flacons commandés dépassera le nombre de flacons produits.
    $\quad$
    Reproduire et compléter sur la copie l’algorithme ci-dessous, écrit en Python, afin qu’après son exécution la variable n contienne le nombre de mois à attendre après le mois de janvier 2020 pour que le nombre potentiel de flacons commandés dépasse le nombre de flacons produits.
    $$\begin{array}{|l|}
    \hline
    \text{n = 0}\\
    \text{f = 2500}\\
    \text{c = 2500}\\
    \text{while $\ldots$ :}\\
    \hspace{1cm}\text{n = $\ldots$}\\
    \hspace{1cm}\text{f = $\ldots$}\\
    \hspace{1cm}\text{c = $\ldots$}\\
    \hline
    \end{array}$$
    $\quad$
  5. De début janvier 2020 à fin décembre 2020, la production globale dépassera-t-elle le nombre de commandes potentielles ? Expliquer votre démarche.
    On rappelle que :

    • Si $\left(u_n\right)$ est une suite arithmétique de premier terme $u_0$, alors, pour tout entier naturel $n$, $$u_0+u_1+\ldots+u_n=(n+1)\dfrac{u_0+u_n}{2}$$
    • Si $\left(v_n\right)$ est une suite géométrique de raison $q\neq 1$, alors, pour tout entier naturel $n$, $$v_0+v_1+\ldots+v_n=v_0\dfrac{1-q^{n+1}}{1-q}$$
      $\quad$

$\quad$

Correction Exercice

  1. On a :
    $\begin{align*} f_1&=f_0+108 \\
    &=2~500+108\\
    &=2~608\end{align*}$
    L’entreprise a produit $2~608$ flacons en février 2020.
    $\quad$
    On a également :
    $\begin{align*} c_1&=\left(1+\dfrac{3,8}{100}\right)c_0\\
    &=1,038\times 2~500\\
    &=2~595\end{align*}$
    $2~595$ flacons ont été commandés en février 2020.
    $\quad$
  2. Pour tout entier naturel $n$ on a $f_{n+1}=f_n+108$. La suite $\left(f_n\right)$ est donc arithmétique de raison $108$ et de premier terme $f_0=2~500$.
    $\quad$
    $\begin{align*} c_{n+1}&=\left(1+\dfrac{3,8}{100}\right)c_n\\
    &=1,038\times c_n\end{align*}$
    La suite $\left(c_n\right)$ est donc géométrique de raison $1,038$ et de premier terme $c_0=2~500$.
    $\quad$
  3. Pour tout entier naturel $n$ on a $f_n=2~500+108n$ et $c_n=2~500\times 1,038^n$.
    $\quad$
  4. On obtient le programme suivant :
    $$\begin{array}{|l|}
    \hline
    \text{n = 0}\\
    \text{f = 2500}\\
    \text{c = 2500}\\
    \text{while c<=f :}\\
    \hspace{1cm}\text{n = n+1}\\
    \hspace{1cm}\text{f = f+108}\\
    \hspace{1cm}\text{c = c*1.038}\\
    \hline
    \end{array}$$
    $\quad$
  5. La production globale sur l’année 2020 est :
    $\begin{align*} F_{11}&=f_0+f_1+\ldots+f_{11} \\
    &=12\times \dfrac{f_0+f_{11}}{2}\\
    &=12\times \dfrac{2~500+2~500+11\times 108}{2}\\
    &=37~128\end{align*}$
    Le nombre total de commandes potentielles sur l’année 2020 est :
    $\begin{align*} C_{11}&=c_0+c_1+\ldots+c_{11} \\
    &=2~500\times \dfrac{1-1,038^{12}}{1-1,038}\\
    &\approx 37~136\end{align*}$
    Ainsi $F_{11}<C_{11}$.
    De début janvier 2020 à fin décembre 2020, la production globale ne dépassera donc pas le nombre de commandes potentielles.
    $\quad$

[collapse]

$\quad$

Les sujets proviennent de la banque nationale de sujets sous licence

 

 

E3C2-Spécialité maths – Suites – 2020

Suites

E3C2 – 1ère

Un jeu vidéo fait évoluer un personnage sur un parcours semé d’obstacles.
Au début du parcours, ce personnage est doté de $1~000$ pions noirs dans son sac et il n’a pas de pion blanc.
Le nombre de pions noirs diminue au cours du jeu.
Le personnage gagne 10 pions blancs par minute jouée.
Chaque partie est chronométrée et dure 45 minutes. Au bout des 45 minutes, la partie s’arrête et le joueur a gagné si le nombre de pions blancs gagnés est supérieur ou égal au nombre de pions noirs du sac.

  1. Étude de l’évolution du nombre de pions blancs
    On note $u_n$ le nombre de pions blancs obtenus au bout de $n$ minutes de jeu.
    Ainsi $u_0 = 0$.
    Déterminer la nature de la suite $\left(u_n\right)$ et en déduire, pour tout entier $n$, l’expression de $u_n$ en fonction de $n$.
    $\quad$
  2. Étude de l’évolution du nombre de pions noirs
    Lucas estime qu’au cours d’une partie, le nombre de ses pions noirs diminue de $2 \%$ par minute. Il voudrait savoir si cette évolution est suffisante pour gagner, ou s’il doit poursuivre son entrainement.
    On note $v_n$ le nombre de pions noirs restant à la $n$-ième minute.
    Ainsi $v_0 = 1~000$.
    a. Justifier que $v_1 = 980$.
    $\quad$
    b. Déterminer la nature de la suite $\left(v_n\right)$ et en déduire, pour tout entier $n$, l’expression de $v$ en fonction de $n$.
    $\quad$
  3. On a calculé les premiers termes des suites $\left(u_n\right)$ et $\left(v_n\right)$ à l’aide d’un tableur. La feuille de calcul est donnée ci-dessous.
    Les termes de la suite $\left(v_n\right)$ ont été arrondis à l’unité.
    Lucas peut-il gagner la partie ?

    $\quad$

$\quad$

Correction Exercice

  1. Pour tout entier naturel $n$ on a donc $u_{n+1}=u_n+10$.
    La suite $\left(u_n\right)$ est donc arithmétique de raison $10$ et de premier terme $u_0=0$.
    Ainsi, pour tout entier naturel $n$, on a $u_n=10n$.
    $\quad$
  2. a. On a :
    $\begin{align*} v_1&=\left(1-\dfrac{2}{100}\right)v_0 \\
    &=0,98\times 1~000\\
    &=980\end{align*}$
    $\quad$
    b. La suite $\left(v_n\right)$ est donc géométrique de raison $0,98$ et de premier terme $v_0=1~000$.
    Pour tout entier naturel $n$ on a donc $v_n=1~000\times 0,98^n$.
    $\quad$
  3. On a donc $u_{45}=450$ et $v_{45}=403$
    Au bout de $45$ minutes, le nombre de pions blancs est bien supérieur au nombre de pions noirs.
    Lucas peut donc gagner la partie.
    $\quad$

[collapse]

$\quad$

Les sujets proviennent de la banque nationale de sujets sous licence

E3C2-Spécialité maths – QCM – 2020

QCM

E3C2 – 1ère

Cet exercice est un questionnaire à choix multiples (QCM). Les cinq questions sont indépendantes. Pour chacune des questions, une seule des quatre réponses est exacte.
Le candidat indiquera sur sa copie le numéro de la question et la lettre correspondant à la réponse exacte. Aucune justification n’est demandée. Une réponse exacte rapporte un point, une réponse fausse ou une absence de réponse ne rapporte ni n’enlève aucun point.

Question 1

$\left(u_n\right)$ est une suite arithmétique telle que $u_4=3$ et $u_{10}=18$. On peut affirmer que :

a. $u_0=7$
b. $u_7=20,5$
c. $u_{12}=23$
d. $u_{14}=-28$

$\quad$

Correction Question 1

$\left(u_n\right)$ est une suite arithmétique de raison $r$.
On a
$\begin{align*} u_{10}=u_4+6r &\ssi 18=3+6r \\
&\ssi 6r=15\\
&\ssi r=2,5\end{align*}$
Donc
$\begin{align*} u_{12}&=u_{10}+2r\\
&=18+2\times 2,5\\
&=23\end{align*}$

Réponse c

$\quad$

[collapse]

$\quad$

Question 2

$2+3+4+\ldots+999+1~000$ est égal à :

a. $500~500$
b. $498~999$
c. $499~000$
d. $500~499$

$\quad$

Correction Question 2

On a
$\begin{align*} S&=2+3+4+\ldots +999+1~000 \\
&=1+2+3+\ldots + 1~000-1\\
&=\dfrac{1~000\times 1~001}{2}-1\\
&=500~499\end{align*}$

Réponse d

$\quad$

[collapse]

$\quad$

Question 3

$\left(v_n\right)$ est la suite géométrique de raison $0,3$ telle que $v_0=-3$. On conjecture que la suite $\left(v_n\right)$ a pour limite :

a. $0$
b. $+\infty$
c. $-\infty$
d. $-3$

$\quad$

Correction Question 3

On a $v_0=-3$, $v_1=-0,9$, $v_2=-0,27$ et $v_3=-0,081$
On peut donc conjecturer que $\lim\limits_{n\to +\infty} v_n=0$.

Réponse a

$\quad$

[collapse]

$\quad$

$\quad$

Question 4

$f$ est la fonction définie sur $\R$ par $f(x)=-2(x+2)^2-3$. On peut affirmer qu’elle est :

a. décroissante sur $]-\infty;+\infty[$
b. décroissante sur $]-2;+\infty[$
c. croissante sur $]-\infty;2[$
d. décroissante sur $]-3;+\infty[$

$\quad$

Correction Question 4

$f$ est une fonction du second degré dont le sommet a pour abscisse $-2$.
Le coefficient principal est $a=-2<0$.
La fonction $f$ est donc strictement décroissante sur l’intervalle $]-2;+\infty[$.

Réponse b

$\quad$

[collapse]

$\quad$

Question 5

L’ensemble des solutions de l’inéquation $x^2-5x+6<0$ est

a. $]-\infty;2[\cup]3;+\infty[$
b. $]-\infty;-1[\cup]6;+\infty[$
c. $]2;3[$
d. $]-1;6[$

$\quad$

Correction Question 5

Le discriminant du polynôme du second degré est :
$\begin{align*}\Delta&=(-5)^2-4\times \times 6\\
&=1\\
&>0\end{align*}$
Les racines du polynômes sont :
$\begin{align*} x_1&=\dfrac{5-\sqrt{1}}{2}\\
&=2\end{align*}$ $\quad$ et $\quad$ $\begin{align*} x_2&=\dfrac{5+\sqrt{1}}{2}\\
&=3\end{align*}$
Le coefficient principal du polynôme est $a=1>0$.
Ainsi les solutions de l’inéquation $x^2-5x+6<0$ est $]2;3[$.

Réponse c

$\quad$

[collapse]

$\quad$

Les sujets proviennent de la banque nationale de sujets sous licence

E3C2-Spécialité maths – QCM – 2020

QCM

E3C2 – 1ère

Ce QCM comprend 5 questions.

Pour chacune des questions, une seule des quatre réponses proposées est correcte.
Les questions sont indépendantes.

Pour chaque question, indiquer le numéro de la question et recopier sur la copie la
lettre correspondante à la réponse choisie.

Aucune justification n’est demandée mais il peut être nécessaire d’effectuer des
recherches au brouillon pour aider à déterminer votre réponse.

Chaque réponse correcte rapporte 1 point. Une réponse incorrecte ou une question
sans réponse n’apporte ni ne retire de point.

Question 1

On considère la loi de probabilité de la variable aléatoire $X$ donnée par le tableau ci-dessous :

$$\begin{array}{|c|c|c|c|c|c|}
\hline
k&-5&0&10&20&50\\
\hline
P(X=k)&0,71&0,03&0,01&0,05&0,2\\
\hline
\end{array}$$
L’espérance de $X$ est :

a. $15$
b. $0,2$
c. $7,55$
d. $17$

$\quad$

Correction Question 1

L’espérance de $X$ est :

$\begin{align*} E(X)&=\small{-5\times 0,71+0\times 0,03+10\times 0,01+20\times 0,05+50\times 0,2} \\
&=7,55\end{align*}$

Réponse c

$\quad$

[collapse]

$\quad$

Question 2

On se place dans un repère orthonormé.
Le cercle de centre A( -2 ; 4) et de rayon 9 a pour équation :

a. $(x+2)^2+(y-4)^2=81$
b. $(x-2)^2+(y+4)^2=81$
c. $(x+2)^2+(y-4)^2=9$
d. $(x-2)^2+(y+4)^2=9$

$\quad$

Correction Question 2

Une équation du cercle est $\left(x-(-2)\right)^2+(y-4)^2=9^2$ soit $(x+2)^2+(y-4)^2=81$.

Réponse a

$\quad$

[collapse]

$\quad$

Question 3

Soit $f$ la fonction définie par $f(x)=ax^2+bx+c$ où $a$, $b$ et $c$ sont des réels.

On considère dans un repère la courbe représentative de $f$ tracée ci-dessous.

On appelle $\Delta$ son discriminant.

On peut affirmer que :

a. $a>0$ ou $c<0$
b. $c$ et $\Delta$ sont du même signe
c. $a<0$ et $c<0$
d. $a<0$ et $\Delta<0$

$\quad$

Correction Question 3

D’après le graphique $a<0$ (la fonction $f$ admet un maximum) et $\Delta>0$ (il y a deux racines)
Les deux racines $x_1$ et $x_2$ sont de signes différents.
Or $ax_1x_2=c$ donc $c>0$
Remarque : On pouvait également lire sur le graphique le fait que $c>0$ puis $f(0)=c$ et graphiquement $f(0)>0$

Réponse b

$\quad$

[collapse]

$\quad$

$\quad$

Question 4

On considère la suite $\left(U_n\right)$ définie par $U_0=-2$ et $U_{n+1}=2U_n-5$.
Un algorithme permettant de calculer la somme $S=U_0+U_1+\ldots+U_{36}$ est :

$\begin{array}{llll}
\textbf{a.}&\begin{array}{|l|}
\hline
\text{U=-2}\\
\text{S=0}\\
\text{Pour i de 1 à 37}\\
\hspace{0.5cm}\text{U$\leftarrow$2U-5}\\
\hspace{0.5cm}\text{S$\leftarrow$S+U}\\
\text{Fin Pour}\\
\hline\end{array}&\textbf{b.}&\begin{array}{|l|}
\hline
\text{U=-2}\\
\text{S=0}\\
\text{Pour i de 1 à 36}\\
\hspace{0.5cm}\text{U$\leftarrow$2U-5}\\
\hspace{0.5cm}\text{S$\leftarrow$S+U}\\
\text{Fin Pour}\\
\hline\end{array}\\\\
\textbf{c.}&\begin{array}{|l|}
\hline
\text{U=-2}\\
\text{S=-2}\\
\text{Pour i de 1 à 37}\\
\hspace{0.5cm}\text{S$\leftarrow$S+U}\\
\hspace{0.5cm}\text{U$\leftarrow$2U-5}\\
\text{Fin Pour}\\
\hline\end{array}&\textbf{d.}&\begin{array}{|l|}
\hline
\text{U=-2}\\
\text{S=-2}\\
\text{Pour i de 1 à 36}\\
\hspace{0.5cm}\text{U$\leftarrow$2U-5}\\
\hspace{0.5cm}\text{S$\leftarrow$S+U}\\
\text{Fin Pour}\\
\hline\end{array}\end{array}$

$\quad$

Correction Question 4

Si la variable $\text{U}$ est transformée avant la variable $\text{S}$ alors $\text{S}$ doit être initialisée à $-2$.
Dans l’algorithme c., quand $\text{i}=1$, la variable $S$, du fait de l’initialisation $S=u_0$, prend la valeur $u_0+u_0$ au lieu de $u_0+u_1$.

Réponse d

$\quad$

[collapse]

$\quad$

Question 5

La suite $\left(U_n\right)$ définie par $U_0=-2$ et $U_{n+1}=2U_n-5$ est :

a. arithmétique mais pas géométrique
b. géométrique mais pas arithmétique
c. ni arithmétique, ni géométrique
d. à la fois arithmétique et géométrique

$\quad$

Correction Question 5

On $U_0=-2$
$\begin{align*} U_1&=2U_0-5\\
&=2\times (-2)-5 \\
&=-9\end{align*}$
$\begin{align*} U_2&=2U_1-5\\
&=2\times (-9)-5\\
&=-23\end{align*}$

Ainsi :

  • $U_1-U_0=-7$ et $U_2-U_1=-14$
    Ces différences ne sont pas égales : la suite n’est pas arithmétique
  • $\dfrac{U_1}{U_0}=\dfrac{9}{2}$ et $\dfrac{U_2}{U_1}=\dfrac{23}{9}$
    Ces quotients ne sont pas égaux : la suite n’est pas géométrique

Réponse c

$\quad$

[collapse]

$\quad$

Les sujets proviennent de la banque nationale de sujets sous licence

E3C2-Spécialité maths – Suites – 2020

Suites

E3C2 – 1ère

Un apiculteur souhaite étendre son activité de production de miel à une nouvelle région.
Au printemps 2019, il achète $300$ colonies d’abeilles qu’il installe dans cette région.
Il consulte les services spécialisés de la région et s’attend à perdre $8\%$ des colonies chaque hiver. Pour maintenir son activité et la développer, il prévoit d’installer $50$ nouvelles colonies chaque printemps, à partir de l’année suivante.

  1. On donne le programme suivant écrit en langage Python :
    $$\begin{array}{|l|}
    \hline
    \text{def algo( ) :}\\
    \hspace{1cm} \text{C = 300}\\
    \hspace{1cm} \text{N = 0}\\
    \hspace{1cm} \text{while C < 400 :}\\
    \hspace{1.5cm} \text{C = C*0.92+50}\\
    \hspace{1.5cm} \text{N = N+1}\\
    \hspace{1cm} \text{return (N)}\\
    \hline
    \end{array}$$
    a. Recopier et compléter en ajoutant des colonnes, le tableau ci-dessous qui
    reproduit l’avancement du programme pas à pas :
    Les valeurs seront arrondies à l’entier le plus proche.
    $$\begin{array}{|c|c|c|c|c}
    \hline
    \text{C}&300&326&\ldots\ldots&\phantom{\ldots\ldots}\\
    \hline
    \text{« C < 400 » ?}&\text{oui}&\text{oui}&\ldots\ldots&\phantom{\ldots\ldots}\\
    \hline
    \end{array}$$
    $\quad$
    b. Quelle est la valeur de $\text{N}$ renvoyée par le programme ?
    Interpréter cette valeur dans le contexte de l’exercice.
    $\quad$

Le nombre de colonies est modélisée par une suite. On note $C_n$ une estimation du nombre de colonies au printemps de l’année 2019 $+ ?$ .

Ainsi $C_0= 300$ est le nombre de colonies au printemps 2019.

On admet que pour tout entier naturel $n$, on a : $$C_{n+1}=0,92C_n+50$$

  1. La suite $\left(C_n\right)$ est-elle arithmétique? La suite $\left(C_n\right)$ est-elle géométrique?
    $\quad$
  2. On admet que $C_n=625-325\times 0,92^n$ pour tout entier naturel $n$.
    L’apiculteur pourra-t-il atteindre les $700$ colonies?
    $\quad$

$\quad$

Correction Exercice

  1. a. On obtient le tableau suivant :
    $$\begin{array}{|c|c|c|c|c|c|c|}
    \hline
    \text{C}&300&326&350&372&392&411\\
    \hline
    \text{« C < 400 » ?}&\text{oui}&\text{oui}&\text{oui}&\text{oui}&\text{oui}&\text{non}\\
    \hline
    \end{array}$$
    $\quad$
    b. Le programme renvoie la valeur $5$.
    Cela signifie que l’apiculteur doit attendre $5$ ans pour avoir au moins $400$ colonies d’abeilles.
    $\quad$
  2. On a $C_0=300$
    $\begin{align*} C_1&=0,92C_0+50\\
    &=0,92\times 300+50\\
    &=326\end{align*}$
    $\begin{align*} C_2&=0,92C_1+50\\
    &=0,92\times 326+50\\
    &=349,92\end{align*}$
    Ainsi $C_1-C_0=26$ et $C_2-C_1=23,92$.
    Ces différences ne sont pas égales : la suite $\left(C_n\right)$ n’est pas arithmétique.
    $\dfrac{C_1}{C_0}\approx 1,087$ et $\dfrac{C_2}{C_1}\approx 1,073$.
    Ces quotients ne sont pas égaux : la suite $\left(C_n\right)$ n’est pas géométrique.
    $\quad$
  3. Pour tout entier naturel $n$ on a $325\times 0,92^n>0$.
    Donc $C_n<625$.
    L’apiculteur ne pourra pas atteindre $700$ colonies.
    $\quad$

[collapse]

$\quad$

Les sujets proviennent de la banque nationale de sujets sous licence

E3C2-Spécialité maths – QCM – 2020

QCM

E3C2 – 1ère

Ce QCM comprend 5 questions.
Pour chacune des questions, une seule des quatre réponses proposées est correcte.
Les questions sont indépendantes.
Pour chaque question, indiquer le numéro de la question et recopier sur la copie la lettre correspondante à la réponse choisie.
Aucune justification n’est demandée mais il peut être nécessaire d’effectuer des recherches au brouillon pour aider à déterminer votre réponse.
Chaque réponse correcte rapporte 1 point. Une réponse incorrecte ou une question sans réponse n’apporte ni ne retire de point.

Question 1

On considère la suite $\left(u_n\right)$ définie par $u_0=100$ et pour tout entier naturel $n$, $u_{n+1}=u_n-\dfrac{13}{100}u_n$.
Quelle est la nature de la suite $\left(u_n\right)$?

a. géométrique de raison $1$
b. arithmétique de raison $-\dfrac{13}{100}$
c. géométrique de raison $1$ et arithmétique de raison $-\dfrac{13}{100}$
d. géométrique de raison $0,87$

Correction Question 1

Pour tout entier naturel $n$ on a :
$\begin{align*} u_{n+1}&=u_n-\dfrac{13}{100}u_n\\
&=0,87u_n\end{align*}$
La suite $\left(u_n\right)$ est donc géométrique de raison $0,87$.

Réponse d

$\quad$

[collapse]

$\quad$

Question 2

On considère la variable aléatoire $X$ qui prend les valeurs $x_i$ pour $i$ entier naturel allant de $1$ à $5$. La loi de probabilité incomplète de la variable aléatoire $X$ est donnée ci-dessous : $$\begin{array}{|c|c|c|c|c|c|}
\hline
X=x_i&-6& -3& 0& 3& x_5\\
\hline
P\left(X=x_i\right)& 0,2& 0,1& 0,2& 0,4& 0,1\\
\hline
\end{array}$$
L’espérance de la variable aléatoire $X$ est égale à $0,7$.
Quelle est la valeur $x_5$ prise par la variable aléatoire $X$?

a. $6$
b. $1$
c. $10$
d. $100$

$\quad$

Correction Question 2

On a :
$\begin{align*} E(X)=0,7&\ssi -6\times 0,2-3\times 0,1+0+3\times 0,4+0,1x_5=0,7 \\
&\ssi-0,3+0,1x_5=0,7\\
&\ssi 0,1x_5=1 \\
&\ssi x_5=10\end{align*}$

Réponse c

$\quad$

[collapse]

$\quad$

Question 3

Soit $f$ la fonction dérivable définie sur $\left]-\dfrac{7}{3};+\infty\right[$ par $f(x)=\dfrac{2x+3}{3x+7}$ et $f’$ sa fonction dérivée.

a. $f'(x)=\dfrac{2}{3}$
b. $f'(x)=\dfrac{23}{(3x+7)^2}$
c. $f'(x)=\dfrac{5}{(3x+7)^2}$
d. $f'(x)=\dfrac{5}{3x+7}$

$\quad$

Correction Question 3

Pour tout réel $x\in \left]-\dfrac{7}{3};+\infty\right[$ on a
$\begin{align*} f'(x)&=\dfrac{2(3x+7)-3(2x+3)}{(3x+7)^2} \\
&=\dfrac{6x+14-6x-9}{(3x+7)^2} \\
&=\dfrac{5}{(3x+7)^2}\end{align*}$

Réponse c

$\quad$

[collapse]

$\quad$

$\quad$

Question 4

De 2017 à 2018, le prix d’un article a augmenté de $10 \%$. En 2019, ce même article a retrouvé son prix de 2017. Quelle a été l’évolution du prix entre 2018 et 2019 ?

a. une baisse de $10 \%$
b. une baisse de plus de $10 \%$
c. on ne peut pas savoir
d. une baisse de moins de $10 \%$

$\quad$

Correction Question 4

On appelle $x$ le pourcentage de diminution appliqué au prix entre 2018 et 2019.
On a ainsi
$\begin{align*} \left(1+\dfrac{10}{100}\right)\times \left(1-\dfrac{x}{100}\right)=1 &\ssi 1,1\left(1-\dfrac{x}{100}\right)=1 \\
&\ssi 1-\dfrac{x}{100}=\dfrac{1}{1,1}\\
&\ssi -\dfrac{x}{100}=\dfrac{1}{1,1}-1\\
&\ssi x=-100\left(\dfrac{1}{1,1}-1\right)\end{align*}$
Ainsi $x\approx 9,1$

Réponse d

$\quad$

[collapse]

$\quad$

Question 5

Soit $\left(u_n\right)$ la suite définie par $u_0=4$ et pour tout entier naturel $n$ par $u_{n+1}=3u_n-5$. On souhaite qu’à la fin de l’exécution de l’algorithme, la valeur contenue dans la variable $u$ soit celle de $u_5$ . Quel algorithme doit-on choisir ?

$\begin{array}{llll}
\textbf{a.}&\begin{array}{|l|}\hline
u=4\\n=0\\\text{For $k$ in range $(5)$ :}\\
\hspace{0.5cm} u=3*n-5\\\hspace{0.5cm}n=n+1\\\hline\end{array}
&\textbf{b.}&\begin{array}{|l|}\hline
u=4\\n=0\\\text{For $k$ in range $(5)$ :}\\
\hspace{0.5cm} u=3*u_n-5\\\hspace{0.5cm}n=n+1\\\hline\end{array}\\\\
\textbf{c.}&\begin{array}{|l|}\hline
u=4\\\text{For $k$ in range $(5)$ :}\\
\hspace{0.5cm} u=3*u-5\\\hline\end{array}
&\textbf{d.}&\begin{array}{|l|}\hline
u=4\\n=0\\\text{While $\pp 5$ :}\\
\hspace{0.5cm} u=3*u-5\\\hspace{0.7cm}n=n+1\\\hline\end{array}\end{array}$

$\quad$

Correction Question 5

Algorithme a : il faudrait avoir $u=3*u-5$
Algorithme b : $u_n$ n’a pas de sens en python
Algorithme d : dans $\text{While }\pp 5$ il manque une variable avant le $\pp$.

Réponse c

$\quad$

[collapse]

$\quad$

Les sujets proviennent de la banque nationale de sujets sous licence

 

E3C2-Spécialité maths – QCM – 2020

QCM

E3C2 – 1ère

Ce QCM comprend 5 questions indépendantes. Pour chacune d’elles, une seule des réponses proposées est exacte.
Indiquer pour chaque question sur la copie la lettre correspondant à la réponse choisie.
Aucune justification n’est demandée.
Chaque réponse correcte rapporte 1 point. Une réponse incorrecte ou une absence de réponse n’apporte ni ne retire de point.

Question 1

L’inéquation $x^2+x+2>0$ :

a. n’a pas de solution
b. a une seule solution
c. a pour ensemble de solution l’intervalle $[1 ; 2]$
d. a pour solution l’ensemble des nombres réels

$\quad$

Correction Question 1

Le discriminant du polynôme du second degré est :
$\begin{align*} \Delta&=1^2-4\times 1\times 2\\
&=-7\\
&<0\end{align*}$

Le coefficient principal est $a=1>0$.
Par conséquent, tous les réels sont solution de l’inéquation $x^2+x+2>0$.

Réponse d

$\quad$

[collapse]

$\quad$

Question 2

Soient $\vec{u}$ et $\vec{v}$ deux vecteurs tels que $\norme{u}=3$, $\norme{v}=2$ et $\vec{u}.\vec{v}=-1$ alors $\left\|\vec{u}+\vec{v}\right\|^2$ est égal à :

a. $11$
b. $13$
c. $15$
d. $25$

$\quad$

Correction Question 2

On a :
$\begin{align*} \vec{u}.\vec{v}&=\dfrac{1}{2}\left(\left\|\vec{u}+\vec{v}\right\|^2-\norme{u}^2-\norme{v}^2\right)\\
\ssi~& -1=\dfrac{1}{2} \left(\left\|\vec{u}+\vec{v}\right\|^2-9-4\right)\\
\ssi~&-2=\left\|\vec{u}+\vec{v}\right\|^2-13\\
\ssi~&\left\|\vec{u}+\vec{v}\right\|^2=11\end{align*}$

Réponse a

$\quad$

[collapse]

$\quad$

Question 3

Soient $A$ et $B$ deux événements d’un univers tels que $P_A(B) = 0,2$ et $P(A) = 0,5$.
Alors la probabilité $P(A\cap B)$ est égale à :

a. $0,4$
b. $0,1$
c. $0,25$
d. $0,7$

$\quad$

Correction Question 3

On a :
$\begin{align*} P_A(B)=\dfrac{P(A\cap B)}{P(A)}&\ssi 0,2=\dfrac{P(A\cap B)}{0,5} \\
&\ssi P(A\cap B)=0,1\end{align*}$

Réponse b

$\quad$

[collapse]

$\quad$

$\quad$

Question 4

Soit $\left(u_n\right)$ une suite arithmétique de terme initial $u_0=2$ et de raison $3$.
La somme $S$ définie par $S=u_0+u_1+\ldots+u_{12}$ est égale à :

a. $45$
b. $222$
c. $260$
d. $301$

Correction Question 4

Pour tout entier naturel $n$ on a donc $u_n=2+3n$

On a :
$\begin{align*} S&=u_0+u_1+\ldots+u_{12} \\
&=(2+3\times 0)+(2+3\times 1)+\ldots +(2+3\times 12) \\
&=2\times 13+3(1+2+\ldots+12)\\
&=26+3\times \dfrac{12\times 13}{2} \\
&=260\end{align*}$

Réponse C

Remarque : Si en cours tu as vu la formule donnant la somme des termes d’une suite arithmétique, tu peux l’utiliser ici:
$\begin{align*} S&=u_0+u_1+ \ldots+u_{12}\\
&=13\times \dfrac{u_0+u_{12}}{2}\\
&=13\times \dfrac{2+38}{2}\\
&=260\end{align*}$

$\quad$

[collapse]

$\quad$

Question 5

Soit $f$ la fonction définie sur l’ensemble des nombres réels par $f(x)=(2x-5)^3$.
Une expression de la dérivée de $f$ est :

a. $3(2x-5)^2$
b. $6(2x-5)^2$
c. $2(2x-5)^2$
d. $2^3$

$\quad$

Correction Question 5

On appelle $g$ la fonction définie sur $\R$ par $g(x)=x^3$.
La fonction $g$ est dérivable sur $\R$ et, pour tout réel $x$, on a $f(x)=g(2x-5)$ et $g'(x)=3x^2$.
Donc $f$ est également dérivable sur $\R$ et, pour tout réel $x$, on a :
$\begin{align*} f'(x)&=2g'(2x-5)\\
&=2\times 3(2x-5)^2\\
&=6(2x-5)^2\end{align*}$

Réponse b

$\quad$

[collapse]

$\quad$

Les sujets proviennent de la banque nationale de sujets sous licence

E3C2-Spécialité maths – QCM – 2020

QCM

E3C2 – 1ère

Cet exercice est un questionnaire à choix multiple (QCM). Pour chaque question, une seule des quatre réponses proposées est exacte. Une bonne réponse rapporte un point. Une mauvaise réponse, une réponse multiple ou l’absence de réponse ne rapporte ni n’enlève aucun point.
Relevez sur votre copie le numéro de la question ainsi que la lettre correspondant à la réponse choisie. Aucune justification n’est demandée.

Question 1

Quelle est la forme factorisée de $f(x)=0,5(x-2)^2-8$?

a. $0,5x^2-2x-6$
b. $0,5(x-6)(x+2)$
c. $0,5(x+10)(x-6)$
d. $0,5(x-10)(x+6)$

$\quad$

Correction Question 1

$\begin{align*} f(x)&=0,5(x-2)^2-8 \\
&=0,5\left[(x-2)^2-16\right]\\
&=0,5\left[(x-2)^2-4^2\right]\\
&=0,5\left[(x-2)-4\right]\left[(x-2)+4\right] \\
&=0,5(x-6)(x+2)\end{align*}$

Réponse b

$\quad$

[collapse]

$\quad$

Question 2

$\left(u_n\right)$ est une suite arithmétique de raison $r = 0,5$ telle que $u_{10} = -4$. Quelle est la valeur du terme $u_2$ ?

a. $8$
b. $0$
c. $-10$
d. $-8$

$\quad$

Correction Question 2

On a $u_{10}=u_2+8r$
Donc $u_2=u_{10}-8r$ soit $u_2=-8$

Réponse d

$\quad$

[collapse]

$\quad$

Question 3

Soit la fonction $f$ définie pour tout $x\neq -2$ par : $f(x)=\dfrac{2x-1}{x+2}$.
Parmi les expressions suivantes, laquelle définit la dérivée $f’$ de la fonction $f$ sur $\R\backslash \lbrace -2\rbrace$ ?

a. $f'(x)=-\dfrac{5}{(x+2)^2}$
b. $f'(x)=\dfrac{5}{(x+2)^2}$
c. $f'(x)=\dfrac{3}{(x+2)^2}$
d. $f'(x)=2$

$\quad$

Correction Question 3

$f$ est dérivable sur $]-\infty;-2[\cup]-2;+\infty[$ en tant que quotient de fonctions dérivables dont le dénominateur ne s’annule pas.
Pour tout réel $x \neq -2$ on a :
$\begin{align*} f'(x)&=\dfrac{2\times(x+2)-1\times(2x-1)}{(x+2)^2} \\
&=\dfrac{2x+4-2x+1}{(x+2)^2} \\
&=\dfrac{5}{(x+2)^2}\end{align*}$

Réponse b

$\quad$

[collapse]

$\quad$

$\quad$

Question 4

On se place dans un repère orthonormé $\Oij$. Laquelle de ces équations est une équation cartésienne de la droite $\Delta$ de vecteur directeur $\vec{u}\begin{pmatrix}-1\\2\end{pmatrix}$ et passant par le point $A(-1;3)$?

a. $2x-y+1=0$
b. $-x+2y-7=0$
c. $x+2y+1=0$
d. $-2x-y+1=0$

$\quad$

Correction Question 4

$\vec{u}\begin{pmatrix}-1\\2\end{pmatrix}$ est un vecteur directeur de $\Delta$.
Une équation de $\Delta$ est donc de la forme $2x+y+c=0$
Le point $A(-1;3)$ appartient à $\Delta$.
Donc $2\times (-1)+3+c=0 \ssi c=-1$.
Une équation de $\Delta$ est donc $2x+y-1=0$.
En multipliant les deux membres par $-1$ on obtient l’équation $-2x-y+1=0$.

Réponse d

$\quad$

[collapse]

$\quad$

Question 5

On se place dans un repère orthonormé $\Oij$. Parmi ces propositions, quelle est l’équation cartésienne du cercle de centre $A(2 ; 4)$ et de rayon $3$ ?

a. $(x-2)^2+(y-4)^2=3$
b. $(x+2)^2+(y+4)^2=9$
c. $x^2+y^2-4x-8y+11=0$
d. $x^2+y^2+11=0$

$\quad$

Correction Question 5

Une équation du cercle est :
$\begin{align*} &(x-2)^2+(y-4)^2=3^2 \\
\ssi~&x^2-4x+4+y^2-8y+16=9 \\
\ssi~&x^2-4x+y^2-8y+11=0\end{align*}$

Réponse c

$\quad$

[collapse]

$\quad$

Les sujets proviennent de la banque nationale de sujets sous licence

E3C2-Spécialité maths – Suites – 2020

Suites

E3C2 – 1ère

Aujourd’hui les chardons (une plante vivace) ont envahi $300$ m² des champs d’une région.
Chaque semaine, la surface envahie augmente de $5 \%$ par le développement des racines, auquel s’ajoutent $15$ m² suite à la dissémination des graines.
Pour tout entier naturel $n$, on note $u_n$ la surface envahie par les chardons, en m$^2$, après $n$ semaines ; on a donc $u_0 = 300$ m$^2$.

  1. a. Calculer $u_1$ et $u_2$.
    $\quad$
    b. Montrer que la suite $\left(u_n\right)$ ainsi définie, n’est ni arithmétique ni géométrique.
    $\quad$
    On admet dans la suite de l’exercice que, pour tout entier naturel $n$, $u_{n+1} = 1,05u_n + 15$.
  2. On considère la suite $\left(v_n\right)$, définie pour tout entier naturel $n$, par : $v_n = u_n + 300$.
    a. Calculer $v_0$, puis montrer que la suite $\left(v_n\right)$ est géométrique de raison $q= 1,05$.
    $\quad$
    b. Pour tout entier naturel $n$, exprimer $v_n$ en fonction de $n$, puis montrer que $u_n = 600 \times 1,05^n-300$.
    $\quad$
  3. Est-il correct d’affirmer que la surface envahie par les chardons aura doublé au bout de $8$ semaines ? Justifier la réponse.
    $\quad$

$\quad$

Correction Exercice

  1. a. On a :
    $\begin{align*} u_1&=\left(1+\dfrac{5}{100}\right)\times u_0+15\\
    &=1,05\times 300+15\\
    &=330\end{align*}$
    et
    $\begin{align*} u_2&=\left(1+\dfrac{5}{100}\right)\times u_1+15\\
    &=1,05\times 330+15\\
    &=361,5\end{align*}$
    $\quad$
    b. On a $u_1-u_0=30$ et $u_2-u_1=31,5$.
    Les différences ne sont pas égales : la suite $\left(u_n\right)$ n’est pas arithmétique.
    $\dfrac{u_1}{u_0}=1,1$ et $\dfrac{u_2}{u_1}\approx 1,092$
    Les quotients ne sont pas égaux : la suite $\left(u_n\right)$ n’est pas géométrique.
    $\quad$
  2. a. On a :
    $\begin{align*} v_0&=u_0+300\\
    &=300+300\\
    &=600\end{align*}$
    Pour tout entier naturel $n$ on a $v_n=u_n+300\ssi u_n=v_n-300$
    $\begin{align*} v_{n+1}&=u_{n+1}+300\\
    &=1,02u_n+15+300\\
    &=1,05\left(v_n-300\right)+315\\
    &=1,05v_n-315+315\\
    &=1,05v_n\end{align*}$
    La suite $\left(v_n\right)$ est donc géométrique de raison $1,05$.
    $\quad$
    b. Pour tout entier naturel $n$ on a $v_n=600\times 1,05^n$.
    Par conséquent :
    $\begin{align*} u_n&=v_n-300\\
    &=600\times 1,05^n-300\end{align*}$
    $\quad$
  3. On a :
    $\begin{align*} u_8&=600\times 1,05^8-300 \\
    &\approx 586,47\end{align*}$
    Par conséquent $u_8<2\times u_0$.
    L’affirmation est donc fausse.
    $\quad$

[collapse]

$\quad$

Les sujets proviennent de la banque nationale de sujets sous licence